LSAT 1 – Section 3 – Question 08

You need a full course to see this video. Enroll now and get started in less than a minute.

Target time: 0:51

This is question data from the 7Sage LSAT Scorer. You can score your LSATs, track your results, and analyze your performance with pretty charts and vital statistics - all with a Free Account ← sign up in less than 10 seconds

Question
QuickView
Type Tags Answer
Choices
Curve Question
Difficulty
Psg/Game/S
Difficulty
Explanation
PT1 S3 Q08
+LR
Flaw or descriptive weakening +Flaw
A
1%
146
B
94%
159
C
2%
146
D
2%
142
E
1%
144
120
126
136
+Easiest 148.102 +SubsectionMedium
This page shows a recording of a live class. We're working hard to create our standard, concise explanation videos for the questions in this PrepTest. Thank you for your patience!

The flaw/descriptively weakening questions, we know this because of the question stem: A flaw in the argument is that the author...

We’re first told that a common procedure for figuring out whether a food additive should be banned is to compare health-related benefits with risks. We’re then given a specific food additive: Yellow Dye No. 5. This could cause allergic reactions for some people. However, the yellow die enhances the drinking experience for some consumers. The argument concludes that because the benefits of enjoying the drink greatly outweighs the allergic reaction risk, the food dye should not be banned.

This argument is internal incoherent. Remember the first sentence we’re giving says that in order to assess whether a food additive should be banned, we should compare it’s health-related benefits to the risks. Enjoyment is not really a “health-related” benefit, it’s just a benefit. Based on what criteria we have in the argument, the conclusion does not logically follow.

Answer Choice (A) is not right because the argument does not imply this; this answer choice is not descriptively accurate. The stimulus implies that enjoyment is a health-related benefit, which isn’t reasonable.

Correct Answer Choice (B) demonstrates the flaw within the argument - the equivocation of enjoyment of a beverage to a health-related benefit.

Answer Choice (C) is descriptively accurate: the stimulus does ignore the possibility that some additives harm people. But this is not a flaw in the argument; this is not even within the scope of what the argument is talking about.

Answer Choice (D) is not good either; remember, we have to accept our premises. The claim about yellow dye has to be accepted.

Answer Choice (E) is debatable on whether it’s relatable. Let’s be generous and say it is. This still isn’t a flaw. What about the some that do not pay attention to the warning labels and are allergic to the dye? Would the enjoyment benefit still outweigh the risks?

Take PrepTest

Review Results

Leave a Reply